Difference between revisions of "2025 AIME II Problems/Problem 15"

m (Solution 1 ('clunky', trial and error))
(Solution 1 ('clunky', trial and error))
Line 41: Line 41:
 
However, we are not finished, trying <math>k=512</math> into the equation <math>(1)</math> from earlier does not give us equality, thus it is an extraneous root. The sum of all <math>k</math> then must be <math>8+32+200=\boxed{240}</math>.
 
However, we are not finished, trying <math>k=512</math> into the equation <math>(1)</math> from earlier does not give us equality, thus it is an extraneous root. The sum of all <math>k</math> then must be <math>8+32+200=\boxed{240}</math>.
  
-lisztepos
+
~ [https://artofproblemsolving.com/wiki/index.php/User:Lisztepos lisztepos]
 +
 
 +
~ Edited by [https://artofproblemsolving.com/wiki/index.php/User:Aoum aoum]
  
 
==See also==
 
==See also==

Revision as of 13:37, 14 February 2025

Problem

Let \[f(x)=\frac{(x-18)(x-72)(x-98)(x-k)}{x}.\]There exist exactly three positive real values of $k$ such that $f$ has a minimum at exactly two real values of $x$. Find the sum of these three values of $k$.

Solution 1 ('clunky', trial and error)

Let $n$ be the minimum value of the expression (changes based on the value of $k$, however is a constant). Therefore we can say that \begin{align*} f(x)-n=\frac{(x-\alpha)^2(x-\beta)^2}{x} \end{align*} This can be done because $n$ is a constant, and for the equation to be true in all $x$ the right side is also a quartic. The roots must also both be double, or else there is an even more 'minimum' value, setting contradiction.

We expand as follows, comparing coefficients:

\begin{align*} (x-18)(x-72)(x-98)(x-k)-nx=(x-\alpha)^2(x-\beta)^2 \\ -2\alpha-2\beta=-18-72-98-k \implies \alpha+\beta=94+\frac{k}{2} \\ \alpha^2+4\alpha \beta +\beta^2=(-18\cdot -72)+(-18\cdot-98)+(-18\cdot-k)+(-72\cdot-98)+(-72\cdot-k)+(-98\cdot-k)=10116+188k \\ (\alpha^2)(\beta^2)=(-18)(-72)(-98)(-k) \implies \alpha \beta=252\sqrt{2k} \\ \end{align*}

Recall $(\alpha+\beta)^2+2\alpha \beta=\alpha^2+4\alpha \beta +\beta^2$, so we can equate and evaluate as follows:

\begin{align} (94+\frac{k}{2})^2+504\sqrt{2k}=10116+188k \tag{1}\\ \end{align} \begin{align*} (47-\frac{k}{4})^2+126\sqrt{2k}=2529 \\ \frac{k^2}{16}-\frac{47}{2}k+126\sqrt{2k}-320=0 \\ \end{align*}

We now have a quartic with respect to $\sqrt{k}$. Keeping in mind it is much easier to guess the roots of a polynomial with integer coefficients, we set $a=\frac{k}{8}$. Now our equation becomes

\begin{align*} 4a^2-188a+504\sqrt{a}-320=0 \\ a^2-47a+126\sqrt{a}-80=0 \\ \end{align*}

If you are lucky, you should find roots $\sqrt{a}=1$ and $2$. After this, solving the resulting quadratic gets you the remaining roots as $5$ and $8$. Working back through our substitution for $a$, we have generated values of $k$ as $(8, 32, 200, 512)$.

However, we are not finished, trying $k=512$ into the equation $(1)$ from earlier does not give us equality, thus it is an extraneous root. The sum of all $k$ then must be $8+32+200=\boxed{240}$.

~ lisztepos

~ Edited by aoum

See also

2025 AIME II (ProblemsAnswer KeyResources)
Preceded by
Problem 14
Followed by
Last Problem
1 2 3 4 5 6 7 8 9 10 11 12 13 14 15
All AIME Problems and Solutions

The problems on this page are copyrighted by the Mathematical Association of America's American Mathematics Competitions. AMC logo.png